Seja $ X \in V_{\omega + \omega} $, seja $ Y = \bigcup X $, por $ (iv) $ temos $ X \in V_{\beta} $ com $ \beta < \omega + \omega $, mas então qualquer elemento dele tem rank $y \in X \implies rank(y) < \beta < \omega + \omega $, nesse caso $ Y \in V_{\omega + \omega} $.